0% found this document useful (0 votes)
28 views10 pages

ECEN280 Spring2020 Homework II - Solutions

Uploaded by

yazanqazzaz
Copyright
© © All Rights Reserved
We take content rights seriously. If you suspect this is your content, claim it here.
Available Formats
Download as PDF, TXT or read online on Scribd
0% found this document useful (0 votes)
28 views10 pages

ECEN280 Spring2020 Homework II - Solutions

Uploaded by

yazanqazzaz
Copyright
© © All Rights Reserved
We take content rights seriously. If you suspect this is your content, claim it here.
Available Formats
Download as PDF, TXT or read online on Scribd
You are on page 1/ 10

American University of Ras Al Khaimah

Electrical, Electronics and Communications Engineering Department


Electric Circuit Analysis I (ECEN 280), Spring 2020
Solutions to Homework II
1. Given the circuit shown in Fig. 1, determine Vx and I0 using nodal analysis.
sup ernode

10 Ω 5V 5Ω
v1 v2 v3

10 Ω I0
20 V 20 Ω 10 Ω VX
6A

Fig. 1. for Problem 1


Solution
Consider nodes 1 and 2.
Applying KCL at Supernode

v1 − 20 v1 v2 v2 − v3
+ + + =0 ⇒ 2v1 − 40 + v1 + 2v2 + 4v2 − 4v3 =0
10 20 10 5
⇒ 3v1 + 6v2 − 4v3 =
40 (1)

Applying KCL at node 3

v3 − v2 v
− 6 + 3 = 0 ⇒ 2v3 − 2v2 − 60 + v3 = 0
5 10
⇒ −2v2 + 3v3 =60 (2)

Supernode Equation:

v1 − v2 =
5 (3)

We have three equations with three unknowns and we can solve it using matrix algebra or using
substitution and successive elimination of variables.
From super node equation (3), we have

4
v1 − v2 =5 ⇒ v1 =v2 + 5 (4)

Substituting (4) into (1) for v1, we have

3v1 + 6v2 − 4v3 = 40 ⇒ 3(v2 + 5) + 6v2 − 4v3 = 40 ⇒ 3v2 + 15 + 6v2 − 4v3 = 40


9v2 − 4v3 =
25 (5)

We can combine (2) and (5) solve the simultaneous equations

=
4 × −2v2 + 3v3 60 =−8v2 + 12v3 240
 ⇒ +
=
3 × 9v2 − 4v3 25 = 27v2 − 12v3 75
19v2 = 315 ⇒ v2 = 315 / 19 = 16.58 V
⇒ v3 =(2v2 + 60) / 3 =(33.16 + 60) / 3 =93.16 / 3 =31.05 V

We can solve for v1 from the supernode equation.

v1 − v2 = 5 ⇒ v1 = v2 + 5 = 16.58 + 5 = 21.58 V

Finally,

• we can see that Vx =v3= 31.05 V


v3 31.05 V
• I= = = 3.11 A
10 Ω
o
10

2. Using mesh analysis find I0 and the voltage v0 in the circuit of Fig. 2.

6Ω

I0 i3
4Ω 2Ω

40 V i1 8Ω v0 i2 4 I0

Fig. 2. for Problem 2

5
Solution
Applying KVL to mesh1

−40 + 4(i1 − i3 ) + 8(i1 − i2 ) =0 ⇒ −40 + 4i1 − 4i3 + 8i1 − 8i2 =0


⇒ 12i1 − 8i2 − 4i3 =
40
⇒ 3i1 − 2i2 − i3 =
10 (1)

Applying KVL to mesh 2

8(i2 − i1 ) + 2(i2 − i3 ) + 4 I 0 =0 ⇒ 8i2 − 8i1 + 2i2 − 2i3 + 4 I 0 =0 but I 0 =i3


⇒ −8i1 + 10i2 + 2i3 =0
⇒ −4i1 + 5i2 + i3 =0 (2)
=
i3 4i1 − 5i2 (3)

Applying KVL to mesh3

4(i3 − i1 ) + 6i3 + 2(i3 − i2 ) =0 ⇒ 4i3 − 4i1 + 6i3 + 2i3 − 2i2 =0


⇒ −4i1 − 2i2 + 12i3 =0
⇒ −2i1 − i2 + 6i3 =0 (4)

Substituting (3) into (1) and (4) for i3, we have

3i1 − 2i2 − i3 =10 ⇒ 3i1 − 2i2 − (4i1 − 5i2 ) =10 ⇒ 3i1 − 2i2 − 4i1 + 5i2 =10
⇒ −i1 + 3i2 =10 (5)

−2i1 − i2 + 6i3 = 0 ⇒ −2i1 − i2 + 6(4i1 − 5i2 ) = 0 = 0 ⇒ −2i1 − i2 + 24i1 − 30i2 = 0


⇒ 22i1 − 31i2 =⇒
0 i2 = i 22
31 1 (6)

Substituting (6) into (5) for i2, we have

−i1 + 3i2 = 10 ⇒ −i1 + 3(22 / 31)i1 = 10 ⇒ (35 / 31)i1 = 10



= i1 310=
/ 35 8.857 A

We can calculate i2 from (6) and i1 from(3) as follows:

=
i2 =
22
i
31 1
22
31 35=
( 310 ) = 6.286 A
220
35

i3 = 4i1 − 5i2 ⇒ i1 = 4( 310


35 ) − 5( 35 ) =
220 140
35 = 4A

Finally,

• I 0= i3= 4 A

6
• v0 =
8(i1 − i2 ) =
= 35 − 35 ) =
8( 310 220 720
35 V=20.571V

−1
 3 −2 −1  i1  10   i1   3 −2 −1 10 
          
 −4 5 1   i2  =
 0  ⇒  i2  =
 −4 5 1   0 
 −2 −1 6   i   0   i   −2 −1 6   0 
  3     3     
>> A=[ 3 -2 -1; -4 5 1; -2 -1 6]

A=

3 -2 -1

-4 5 1

-2 -1 6

>> b=[10 0 0].'

b=

10

>> i=inv(A)*b

i=

8.8571

6.2857

4.0000

7
3. Use the superposition principle to find the current i0 and the voltage v0 in the circuit shown in Fig.
3.

5Ω

10 A 2i0 60 V
30 V

Fig. 3 for Question 3


Solution:
We consider the contribution of the 10 A and the 30 V sources separately taking one at a time. We live
the dependent sources intact while considering each independent source.

Let the contribution of the 10 A source is V1 and the contribution of the 30 V source is V2 to the total of
v0.

v0
v0 = V1 + V2 ⇒ i0 =
10 Ω

Case 1: Consider contribution of the 10 A source. We short circuit the 30 V source and analyze the circuit
to get V1 as shown in the circuit below.
i1 5Ω

10 A 2i1

Applying KCL at node a:

va  va − vb 
10 − −  =0 ⇒ 400 − va − 8va + 8vb =0
40  5 
−9va + 8vb = −400 (1)

8
Applying KCL at node b:

 v − va  vb v −v
2i1 −  b − =0 ⇒ but i1 = a b
 5  20 5
 v − v   v − va  vb
⇒ 2 a b  −  b  − 20 =0 ⇒ 8(va − vb ) − 4(vb − va ) − vb =0
 5   5 
⇒ 8va − 8vb − 4vb + 4va − vb =0 ⇒ 12va − 13vb =0 ⇒ va =(13 / 12)vb (2)

Substituting (2) in (1) for va

−9va + 8vb =
−400 ⇒ −9(13 / 12vb ) + 8vb =
−400
⇒ −(117 / 12)vb + 8vb = −400 ⇒ −(21 / 12)vb = −400 ⇒ vb = 228.57 V
=
⇒ va (13 /=
12)vb 247.62V

Therefore the contribution due to the 12 A current source is


V1 = va − vb = 247.62 − 228.57 = 19.05 V (3)

Case 2: Consider contribution of the 30 V source. We open circuit the 10 A current source and analyze
the circuit to get V2 as shown in the circuit below.

i2 5 Ω

2i2 30 V
i2

Applying KCL at node c:

vc  v + 30  v
− + 2i2 −  c  =
0 ⇒ but i2 =− c
45  20  45
v 2v  v + 30 
⇒ − c − c − c = 0 ⇒ −4vc − 8vc − 9vc = 270
45 45  20 
vc 12.857
⇒ −21vc = 270 ⇒ vc = −12.857 V ⇒ i2 = − = = 0.2857 A
45 45
V=
2 5=
i2 1.429V

Therefore, using the superposition property

20.48 V
v0 = V1 + V2 = 19.05 V+1.429 V=20.48V ⇒ i0 = = 4.10 A
5Ω

9
4. For circuit of Fig. 4, find the Thevenin equivalent circuit between the terminals a and b.

40 Ω

40 V
20 Ω 40 Ω

20 Ω

20 Ω 20 Ω

40 V

Fig. 4. for Problem 4


Solution: We need to fin Vth and Rth for the circuit between terminals a and b. To find Rth the two
independent voltage sources will be short-circuited and the independent current source will be open
circuited. The two 40Ω resistors are in parallel giving:
−1
 1 1 
40 Ω / /40Ω=  +  = 20Ω
 40 40 
The circuit is re-drawn as follows

20 Ω 20 Ω

20 Ω
20 Ω
20 Ω

The center Y- connected resistors can be converted to ∆ as follows:

20 × 20 + 20 × 20 + 20 × 20 3 × 20 × 20
∆ resistor = = =60 Ω
20 20

10
60 Ω

20 Ω
60 Ω 60 Ω

20 Ω 20 Ω

From the above circuit, the equivalent Thevenin resistance between the terminals a and b can be easily
determined.

RTh = [ (20 Ω / /60 Ω) + (20 Ω / /60 Ω) ] / /60 Ω


  1 1  
−1

= 2 ×  +   / /60 Ω= [ 2 × 15Ω] / /60 Ω= 30Ω / /60Ω


  20 60  
−1
 1 1  30 × 60 1800 Ω
=
 +  = = =
20 Ω
 30 60  (30 + 60) 90

To find Vth, we can change the 5A current source to a voltage source using source transformation.
Similarly, we firstly change the 40 V voltage source with the 40 Ω series resistor to a 1 A current source
and 40 Ω parallel resistor, then find the equivalent resistance of the two 40 Ω parallel resistors that gives
20 Ω resistor and finally change back the 1 A current source to a voltage source. We can re-draw the
circuit as follows:

20 V
20 Ω 20 Ω

20 Ω 20 Ω
20 Ω

40 V 100 V

Now, we can use KVL to the loops 1 and 2 to find i1 and i2 then the voltages Va and Vb.

Applying KVL for loop 1:

−40 + 20i1 + 20i1 + 20(i1 − i2 ) + 100 =


0 ⇒ 60i1 − 20i2 =
−60 ⇒ 3i1 − i2 =
−3 (1)

11
Applying KVL for loop 2:

−100 + 20(i2 − i1 ) + 20i2 − 20 + 20i2 = 0 ⇒ −20i1 + 60i2 = 120 ⇒ −i1 + 3i2 = 6 (2)

We can combine (1) and (2) to solve for i1 and i2.

3 × 3i1 − i2 =−3

+ −i1 + 3i2 = 6
3
8i1 =−3 ⇒ i1 =− =−0.375 A ⇒ i2 =3 + 3i1 =3 − 1.125 =1.875 A
8
The voltage at points a and b are calculated as follows:

va = 40 − 20i1 = 40 − 20(−0.375) = 40 + 7.5 = 47.5 V (3)


vb =
20i2 =
20 × 1.875 =
37.5 V (4)

Therefore , VTh = va − vb = 47.5 − 37.5 = 10 V

5. Calculate the output voltage vo in the op-amp circuit shown in Fig. 5.

100 kΩ

20 kΩ 20 kΩ
100 V

10 kΩ 10 kΩ

Fig. 5. for Problem 5

Solution:
Consider the two nodes v1 and v2 as shown below. Since the op-amp is an ideal circuit v1=v2 and i1=i2
=0.

12
100 kΩ
v1
20 kΩ 20 kΩ i1
100 V

10 kΩ 10 kΩ i2

v2

Fig. 5. for Problem 5

Applying KCL at node 1

v1 − v0 v1 v1 − 100
+ + =0 ⇒ v1 − v0 + 5v1 + 5v1 − 500 =0
100 20 20
⇒ v0 = 11v1 − 500 (1)

Looking at the circuit, node 1 and node 2 are at the same potential, that is v1=v2
Applying KCL at node 2

v2 v2 − 100
+ =0 ⇒ 2v2 =100 ⇒ v2 =50 V =v1 (2)
10 10

Substituting 2 in (1) for v1, we have

v0 =11v1 − 500 =11 × 50 − 500 =550 − 500 =50 V

13

You might also like